Difference between revisions of "2017 AMC 12A Problems/Problem 21"

(Created page with "==Problem== A set <math>S</math> is constructed as follows. To begin, <math>S = \{0,10\}</math>. Repeatedly, as long as possible, if <math>x</math> is an integer root of some...")
 
Line 8: Line 8:
 
\qquad\textbf{(D)}\ 9
 
\qquad\textbf{(D)}\ 9
 
\qquad\textbf{(E)}\ 11</math>
 
\qquad\textbf{(E)}\ 11</math>
 +
 +
==See Also==
 +
{{AMC12 box|year=2017|ab=A|num-b=20|num-a=22}}
 +
{{MAA Notice}}

Revision as of 18:24, 8 February 2017

Problem

A set $S$ is constructed as follows. To begin, $S = \{0,10\}$. Repeatedly, as long as possible, if $x$ is an integer root of some polynomial $a_{n}x^n + a_{n-1}x^{n-1} + ... + a_{1}x + a_0$ for some $n\geq{1}$, all of whose coefficients $a_i$ are elements of $S$, then $x$ is put into $S$. When no more elements can be added to $S$, how many elements does $S$ have?

$\textbf{(A)}\ 4 \qquad \textbf{(B)}\ 5 \qquad\textbf{(C)}\ 7 \qquad\textbf{(D)}\ 9 \qquad\textbf{(E)}\ 11$

See Also

2017 AMC 12A (ProblemsAnswer KeyResources)
Preceded by
Problem 20
Followed by
Problem 22
1 2 3 4 5 6 7 8 9 10 11 12 13 14 15 16 17 18 19 20 21 22 23 24 25
All AMC 12 Problems and Solutions

The problems on this page are copyrighted by the Mathematical Association of America's American Mathematics Competitions. AMC logo.png